Chapter 3
Chapter 3
Chapter 3
√
We know that
√ 2 is not a rational number, but we can find rational numbers as close as
we wish to 2. For instance, the sequence of rationals
Thus it seems that we can create a square root of 2 by taking a “limit” of a sequence of
rational numbers. This is how we shall construct the reals.
Definition 1 (Sequence). A sequence of real numbers or a sequence in R is a mapping
f : N → R. We write xn for f (n), n ∈ N and it is customary to denote a sequence as hxn i
or (xn ) or {xn }.
Example 1. There are different ways of expressing a sequence. For example:
(1) Constant sequence: (a, a, a, . . .), where a ∈ R
(4) Sequence defined recursively: (xn ), where x1 = 4 and xn+1 = 2xn − 5 for all n ∈ N
Convergence: What does it mean?
2. ( n1 )
3. ((−1)n n1 )
4. (1, 2, 1, 2, . . .)
√
5. ( n)
6. ((−1)n (1 − n1 ))
7. (n2 − 1)
Definition 2 (Convergent sequence). A sequence (xn ) is said to be convergent if there
exists ` ∈ R such that for every ε > 0, there exists n0 ∈ N satisfying |xn − `| < ε for all
n ≥ n0 . We say that ` is a limit of (xn ).
Notation: We write lim xn = ` or xn → `.
n→∞
Proof. Let (xn ) be a convergent sequence. Assume that lim xn = `1 and lim xn = `2 .
n→∞ n→∞
We claim that `1 = `2 . To see this, by way of contradiction assume that `1 6= `2 . Then
ε = |`1 −`
3
2|
> 0. By definition of convergence of a sequence, there are positive integers n1
and n2 such that
Solution. Let ε > 0. By using the Archimedean property, we can find a positive integer
n0 such that n0 · ε > 1, that is, ε > n10 . Now, for all n ≥ n0 , we have
1 1 1
| − 0| = ≤ < ε.
n n n0
1
This proves that lim = 0.
n→∞ n
Example 3. Consider the sequence (xn ) where xn = (−1)n . The terms of the sequence
are −1, 1, −1, 1, −1, 1, . . .. It is intuitively clear that this sequence does not approach to
any real number. Therefore, the sequence does not converge. We now establish this fact
by using the definition.
Solution. By way of contradiction assume that the given sequence converges to `. Then
for ε = 21 , there exists a natural number n0 such that
1
|(−1)n − `| < for all n ≥ n0 .
2
1
This gives |1 − `| < 2
and |1 + `| < 12 . Now, using triangle inequality we have
2 = |1 + 1| = |1 − ` + ` + 1| ≤ |1 − `| + |1 + `| < 1,
Thus, the set {xn0 , xn0 +1 , xn0 +2 , . . .} is bounded from below by ` − 1 and bounded from
above by ` + 1. Using the triangle inequality, we have
Now, there are only finitely many elements left, namely x1 , x2 , . . . , xn0 −1 . Let
(d) xn yn → xy.
xn x
(e) yn
→ y
if yn 6= 0 for all n ∈ N and y 6= 0.
−3n 2
Example 5. The sequence ( 3n2n2 +5n+3 ) is convergent with limit 32 .
3
2n2 −3n 2− n 1
Solution. We have 3n2 +5n+3
= 5
3+ n + 3 for all n ∈ N. Since n
→ 0, the limit rules for
n2
algebraic operations on sequences imply that the given sequence is convergent with limit
2−0
3+0+0
= 23 .
√ √
Example 6. The sequence ( n + 1 − n) is convergent with limit 0.
√ √ √1
Solution. For all n ∈ N, n + 1 − n = √n+1+ 1 √
n
= √ n
1
. Since n1 → 0, the limit
1+ n +1
rules for algebraic operations on sequences imply that the given sequence is convergent
0
with limit √1+0+1 = 0.
Theorem 4 (Sandwich theorem). Let (xn ), (yn ), (zn ) be sequences such that xn ≤ yn ≤ zn
for all n ∈ N. If both (xn ) and (zn ) converge to the same limit `, then (yn ) also converges
to `.
Proof. Let ε > 0. Since xn → `, so there exists a positive integer n1 such that |xn − `| < ε
for all n ≥ n1 . Similarly, as zn → `, so there exists a positive integer n2 such that
|zn − `| < ε for all n ≥ n2 . Let n0 = max{n1 , n2 }. Then,
` − ε < xn < ` + ε and ` − ε < zn < ` + ε for all n ≥ n0 .
Using the given fact that xn ≤ yn ≤ zn for all n ∈ N, we obtain
` − ε < xn ≤ yn ≤ zn < ` + ε for all n ≥ n0 .
This proves that |yn − `| < ε for all n ≥ n0 , and hence yn → `.
cos n −1 cos n 1
Example 7. Consider the sequence . Since ≤ ≤ for all n ∈ N, so
n n n n
cos n
by applying Sandwich theorem we find that lim = 0.
n→∞ n
1
Example 8. If α > 0, then the sequence (α n ) converges to 1.
1
Solution. We first assume that α ≥ 1 and let xn = α n − 1 for all n ∈ N. Then xn ≥ 0
and α = (1 + xn )n = 1 + nxn + n(n−1)
2!
x2n + · · · + xnn > nxn for all n ∈ N. So 0 ≤ xn < αn
α
for all n ∈ N. Since n → 0, by Sandwich theorem, it follows that xn → 0. Consequently
1 1 1
α n → 1. If α < 1, then α1 > 1 and as proved above, ( α1 ) n → 1. It follows that α n → 1.
Alternative proof: We first assume that α ≥ 1. For each n ∈ N, applying the A.M. ≥ G.M.
1
inequality for the numbers 1, ..., 1, α (1 is repeated n − 1 times), we get 1 ≤ α n ≤ 1 + α−1
n
.
α−1 1
Since n → 0, by Sandwich theorem, it follows that α → 1. The case for α < 1 is same
n
Proof. We know that between two real numbers, there is a rational number. For each
n ∈ N, consider the real numbers r − n1 and r + n1 . Let xn be a rational number such
that r − n1 < xn < r + n1 . Then (xn ) is a sequence of rational numbers, and by Sandwich
theorem (xn ) converges to r.
Divergent sequence:
• If (xn ) and (yn ) both diverge to ∞, then the sequences (xn + yn ) and (xn yn ) also
diverge to ∞.
Monotone sequence:
Theorem 6. If (xn ) is increasing and not bounded above then (xn ) diverges to ∞. If (xn )
is decreasing and not bounded below then (xn ) diverges to −∞.
Theorem 7 (Monotone convergence theorem). Let (xn ) be a sequence of real numbers.
(a) If (xn ) is increasing and bounded above then (xn ) converges to sup{xn : n ∈ N}.
(b) If (xn ) is decreasing and bounded below then (xn ) converges to inf{xn : n ∈ N}.
(c) A monotonic sequence converges if and only if it is bounded.
Proof. We only prove (a). Since (xn ) is increasing, so it is bounded below by x1 . Also,
(xn ) is bounded above and hence (xn ) is bounded. Let s = sup{xn : n ∈ N}. We claim
that xn → s. To prove this, let ε > 0. Then s − ε is not an upper bound and so there
exists some n0 such that s − ε < xn0 . Since (xn ) is increasing so xn ≥ xn0 for all n ≥ n0 .
Therefore
s − ε < xn0 ≤ xn ≤ s < s + ε for all n ≥ n0 .
This proves that xn → s.
Example 14. Let x1 = 1 and xn+1 = 13 (xn + 1) for all n ∈ N. Then the sequence (xn ) is
convergent and lim xn = 21 .
n→∞
Solution. For all n ∈ N, we have xn+1 − xn = 13 (1 − 2xn ). Also, x1 > 21 and if we assume
that xk > 21 for some k ∈ N, then xk+1 = 13 (xk + 1) > 13 ( 12 + 1) = 12 . Hence by the
principle of mathematical induction, xn > 12 for all n ∈ N. So (xn ) is bounded below.
Again, from above, we get xn+1 − xn < 0 for all n ∈ N ⇒ xn+1 < xn for all n ∈ N ⇒ (xn )
is decreasing. Therefore (xn ) is convergent. Let ` = lim xn . Then lim xn+1 = ` and
n→∞ n→∞
since xn+1 = 31 (xn + 1) for all n ∈ N, we get ` = 13 (` + 1) ⇒ ` = 12 .
Example 15. The sequence ((1 + 1/n)n ) is convergent.
Solution. Let an = (1 + 1/n)n . Then
n k
X n 1 n 1 n(n − 1) 1 n(n − 1) · · · 2 · 1 1
an = =1+ · + · 2 + ··· + · n
k=0
k n 1 n 2! n n! n
1 1 1 1 2 n−1
=1+1+ 1− + ··· + 1− 1− ··· 1 − .
2! n n! n n n
Similarly, we have
1 1 1 1 2 n−1
an+1 =1+1+ 1− + ··· + 1− 1− ··· 1 −
2! n+1 n! n+1 n+1 n+1
1 1 2 n
+ 1− 1− ··· 1 − .
(n + 1)! n+1 n+1 n+1
Note that the expression for an contains n + 1 terms, while that for an+1 contains n + 2
terms. Moreover, each term appearing in an is less than or equal to the corresponding
term in an+1 , and an+1 has one more positive term. Therefore, we have
2 ≤ a1 < a2 < · · · < an < an+1 < · · · ,
so that the sequence (an ) is increasing. For n > 1, we have
1 1 1 1
2 < an < 1 + 1 + + + · · · + n−1 = 2 + 1 − n−1 < 3.
2 22 2 2
By Monotone convergence theorem, the sequence (an ) converges to a real number that
lies between 2 and 3. We define the number e to be the limit of this sequence.
xn+1
Theorem 8. Let xn 6= 0 for all n ∈ N and let L = lim xn
exist.
n→∞
Proof. Note that L ≥ 0. Let r be such that L < r < 1. Let ε = r − L. Then ε > 0, and
hence there is some n0 ∈ N such that
|xn+1 |
0≤ < L + ε = r for all n ≥ n0 .
|xn |
Example 18. Think of some divergent sequences and their convergent subsequences.
• If (xn ) has two subsequences converging to two different limits, then (xn ) cannot be
convergent.
Example 20. Let (xn ) be a sequence in R. Then (x2n ) and (x2n−1 ) are two subsequences
of (xn ). Suppose that x2n → ` ∈ R and x2n−1 → `. Then xn → `.
Solution. Let ε > 0. Since x2n → ` and x2n−1 → `, there exist n1 , n2 ∈ N such that
|x2n − `| < ε for all n ≥ n1 and |x2n−1 − `| < ε for all n ≥ n2 . Taking n0 = max{2n1 , 2n2 −
1} ∈ N, we find that |xn − `| < ε for all n ≥ n0 . Hence xn → `.
Proof. Let (xn ) be a sequence of real numbers. A term xp is called a peak in (xn ) if
xp > xm for all m > p. That is, a peak in (xn ) is a term which is greater than all the
succeeding terms. Let P be the set of all the peaks of (xn ). We now consider the following
two cases:
• P is finite: Note that in this case P can be empty also. Let p1 < p2 < · · · < p` so
that xp1 , xp2 , . . . , xp` are the only peaks of (xn ). Let n1 > p` . Then xn1 is not a peak.
Hence there is some n2 ∈ N such that n2 > n1 and xn1 ≤ xn2 . Again, since n2 > p`
so xn2 is not a peak. Hence, there is some n3 > n2 such that xn2 ≤ xn3 . In this
way, using the principle of mathematical induction, we have an increasing sequence
n1 < n2 < n3 < · · · < nk < · · · in N such that xn1 ≤ xn2 ≤ xn3 ≤ · · · < xnk < · · · .
This proves that (xnk ) is an increasing subsequence of (xn ).
• P is infinite: In this case, we have p1 < p2 < · · · < pk < · · · so that xp1 , xp2 , . . . , xpk , . . .
are the peaks of (xn ). Clearly, xp1 > xp2 > . . . > xpk > . . .. Hence, (xpk ) is a de-
creasing sequence of (xn ).
Theorem 11 (Bolzano-Weierstrass Theorem). Every bounded sequence in R has a con-
vergent subsequence.
Proof. Let (xn ) be a sequence of real numbers. By the previous theorem, (xn ) has a
monotone subsequence, say (xnk ). Since (xn ) is bounded, so (xnk ) is also bounded. By
the Monotone convergence theorem, (xnk ) is convergent.
Cauchy sequence:
Definition 7 (Cauchy sequence). A sequence (xn ) is called a Cauchy sequence if for each
ε > 0, there exists n0 ∈ N such that |xm − xn | < ε for all m, n ≥ n0 .
Theorem 12. Every Cauchy sequence is bounded.
Theorem 13 (Cauchy’s criterion for convergence). A sequence in R is convergent if and
only if it is a Cauchy sequence.
Proof. Let (xn ) be convergent and xn → `. Let ε > 0. Then there is some n0 ∈ N such
that |xn − `| < ε/2 for all n, m ≥ n0 . Now,
Hence, (xn ) is a Cauchy sequence. Conversely, suppose that (xn ) is a Cauchy sequence.
Then (xn ) is bounded, and by Bolzano Weierstrass theorem, (xn ) has a convergent sub-
sequence, say (xnk ). Suppose that xnk → `. We claim that xn → `. To prove this, let
ε > 0. Since (xn ) is Cauchy, so there is some n0 ∈ N such that |xn − xm | < ε/2 for all
n, m ≥ n0 . Also, since xnk → `, there is k0 ∈ N such that |xnk − `| < ε/2 for all k ≥ k0 .
Let j = max{k0 , n0 }. Since nj ≥ j, so nj ≥ n0 . Also, j ≥ k0 . Therefore
Let (xn ) be a bounded sequence. Let y1 = sup{x1 , x2 , . . .}, y2 = sup{x2 , x3 , . . .}, and
so on. That is, for n ∈ N,
Let A and B be two nonempty subsets of R such that A ⊆ B. Then clearly, sup(A) ≤
sup(B) and inf(B) ≤ inf(A). Hence,
y1 ≥ y2 ≥ y3 ≥ · · · .
Since (xn ) is bounded so the sequence (yn ) is bounded below. By Monotone convergence
theorem, (yn ) is convergent and converges to the infimum of {y1 , y2 , . . .}. The limit of
the sequence (yn ) is called the limit superior of the sequence (xn ), and is denoted by
lim sup xn . Thus,
lim sup xn := lim yn = inf sup xk .
n→∞ n k≥n
Similarly, let z1 = inf{x1 , x2 , . . .}, z2 = inf{x2 , x3 , . . .}, and so on. That is, for n ∈ N,
Example 25. Consider the sequence (xn ), where xn = n1 . Clearly, for any n, yn = sup{ k1 :
k ≥ n} = n1 and zn = inf{ k1 : k ≥ n} = 0. Hence, lim sup xn = lim yn = lim n1 = 0 and
n→∞ n→∞
lim inf xn = 0.
(2) If an ≤ bn for all n ∈ N, then lim sup an ≤ lim sup bn and lim inf an ≤ lim inf bn .
(3) lim sup(an +bn ) ≤ lim sup an +lim sup bn and lim inf(an +bn ) ≥ lim inf an +lim inf bn .
Proof. Let ` = lim an . Let ε > 0. Then there is some n0 ∈ N such that
Theorem 16. Let (an ) be a bounded sequence. If lim sup an = lim inf an , then (an ) is
convergent and lim an = lim sup an .
n→∞
Proof. Let lim sup an = lim inf an = `. Let yn = sup{ak : k ≥ n} and zn = inf{ak : k ≥
n}. Then we have lim sup an = lim yn = ` and lim inf an = lim zn = `. Let ε > 0. Then
there are positive integers n1 and n2 such that
Hence, lim an = `.
Alternative proof: Equivalently, we can directly apply Sandwich Theorem. We have
zn ≤ an ≤ yn for all n. Since zn → ` and yn → `, by using Sandwich Theorem, we have
an → `.